Difference between revisions of "2001 AIME I Problems/Problem 2"
m (→Solution 2) |
Bluelinfish (talk | contribs) m (→Solution 2) |
||
Line 12: | Line 12: | ||
==Solution 2== | ==Solution 2== | ||
− | Since this is a weighted average problem, the mean of <math>S</math> is <math>\frac{13}{27}</math> as far from <math>1</math> as it is from <math>2001</math> Thus, the mean of <math>S</math> is | + | Since this is a weighted average problem, the mean of <math>S</math> is <math>\frac{13}{27}</math> as far from <math>1</math> as it is from <math>2001</math>. Thus, the mean of <math>S</math> is |
<math>1 + \frac{13}{13 + 27}(2001 - 1) = \boxed{651}</math>. | <math>1 + \frac{13}{13 + 27}(2001 - 1) = \boxed{651}</math>. | ||
Revision as of 07:40, 27 August 2020
Contents
Problem
A finite set of distinct real numbers has the following properties: the mean of is less than the mean of , and the mean of is more than the mean of . Find the mean of .
Solution
Let be the mean of . Let be the number of elements in . Then, the given tells us that and . Subtracting, we have We plug that into our very first formula, and get:
Solution 2
Since this is a weighted average problem, the mean of is as far from as it is from . Thus, the mean of is .
See Also
2001 AIME I (Problems • Answer Key • Resources) | ||
Preceded by Problem 1 |
Followed by Problem 3 | |
1 • 2 • 3 • 4 • 5 • 6 • 7 • 8 • 9 • 10 • 11 • 12 • 13 • 14 • 15 | ||
All AIME Problems and Solutions |
The problems on this page are copyrighted by the Mathematical Association of America's American Mathematics Competitions.